Finite Integral Domains .... Adkins & Weintraub, Propn 1.5

In summary, the author proves that the equation ##ax = 1## is solvable for every ##a \neq 0## by showing that the mapping ##\phi_a## is bijective, and that this conclusion leads to the fact that ##R## is a field. This is similar to the proof that every element of a finite group has a finite order, and relies on the properties of integral domains.
  • #1
Math Amateur
Gold Member
MHB
3,990
48
I am reading "Algebra: An Approach via Module Theory" by William A. Adkins and Steven H. Weintraub ...

I am currently focused on Chapter 2: Rings ...

I need help with an aspect of the proof of Proposition 1.5 ... ...

Proposition 1.5 and its proof read as follows:
A&W - Proposition 1.5 ... .png
At the end of the above proof from Adkins and Weintraub we read the following:

" ... ... and hence ##\phi_a (R) = R##. In particular, the equation ##ax = 1## is solvable for every ##a \neq 0## and ##R## is a field. ... ... "
Can someone please explain to me how the conclusion that "the equation ##ax = 1## is solvable for every ##a \neq 0## and ##R## is a field" follows from the arguments preceding it ...

Basically I do not understand how the arguments before this statement lead to the conclusion ...Help will be much appreciated ...

Peter
 

Attachments

  • A&W - Proposition 1.5 ... .png
    A&W - Proposition 1.5 ... .png
    20.2 KB · Views: 493
Physics news on Phys.org
  • #2
The author showed, that ##\phi_a## is bijective, because it is injective and surjective: ##\phi_a(R)=R##.
That means ##a \cdot x = \phi_a(x) = 1## has exactly one solution ##x \in \phi_a^{-1}(\{1\})##.
 
Last edited:
  • Like
Likes Math Amateur
  • #3
fresh_42 said:
The author showed, that ##\phi_a## is bijective, because it is injective and surjective: ##\phi_a(R)=R##.
That means ##a \cdot x = \phi_a(x) = 1## has exactly one solution ##x \in \phi_a^{-1}(\{1\})##.
Thanks fresh_42 ...

Reflecting on what you have said ...

But ... hmm ... yes ... seems right ...

Thanks again ...

Peter
 
  • #4
Integral domains and Fields has a nice proof that every finite integral domain is a field. It is much like the proof that every element of a finite group has a finite order. One takes some nonzero element a and multiplies it by itself until one comes with two exponents m and n such that am = an. Then,

am - an = an * (am-n - 1) = 0

From the definition of integral domain, either an = 0 or am-n - 1 = 0. In the first case, an = a * an-1, and if it equals 0, then either a or a an-1 equals 0. Continuing for an-1, we find that a = 0, contrary to our condition for a.

But if am-n - 1 = 0, then a * am-n-1 = 1. Thus, a has a multiplicative inverse, and thus every finite integral domain is a field.
 
  • Like
Likes Math Amateur

1. What is a finite integral domain?

A finite integral domain is a commutative ring with unity that has a finite number of elements and satisfies the property that every non-zero element has a multiplicative inverse.

2. How is a finite integral domain different from a finite field?

A finite field is also a commutative ring with unity and a finite number of elements, but it also has the property that every non-zero element has an additive inverse. This means that every element in a finite field can be multiplied by another element to get the additive identity, while in a finite integral domain, this is only true for non-zero elements.

3. Can a finite integral domain have zero divisors?

No, a finite integral domain cannot have zero divisors. This is because every non-zero element has a multiplicative inverse, so if we multiply any two non-zero elements, their product will always be non-zero.

4. How is the concept of a finite integral domain used in mathematics?

Finite integral domains are important in abstract algebra and number theory. They are used in the study of rings, fields, and group theory, and have applications in cryptography and coding theory.

5. Can a finite integral domain have infinite elements?

No, by definition, a finite integral domain has a finite number of elements. If a commutative ring with unity has an infinite number of elements, it cannot be a finite integral domain.

Similar threads

  • Linear and Abstract Algebra
Replies
2
Views
1K
  • Linear and Abstract Algebra
Replies
7
Views
1K
  • Linear and Abstract Algebra
Replies
2
Views
1K
  • Linear and Abstract Algebra
Replies
3
Views
1K
Replies
3
Views
1K
Replies
3
Views
1K
  • Linear and Abstract Algebra
Replies
16
Views
2K
  • Linear and Abstract Algebra
Replies
1
Views
987
Replies
1
Views
989
Replies
2
Views
1K
Back
Top